Stay ahead of learning milestones! Enroll in a class over the summer!

G
Topic
First Poster
Last Poster
Circles tangent at orthocenter
Achillys   62
N an hour ago by Rayvhs
Source: APMO 2018 P1
Let $H$ be the orthocenter of the triangle $ABC$. Let $M$ and $N$ be the midpoints of the sides $AB$ and $AC$, respectively. Assume that $H$ lies inside the quadrilateral $BMNC$ and that the circumcircles of triangles $BMH$ and $CNH$ are tangent to each other. The line through $H$ parallel to $BC$ intersects the circumcircles of the triangles $BMH$ and $CNH$ in the points $K$ and $L$, respectively. Let $F$ be the intersection point of $MK$ and $NL$ and let $J$ be the incenter of triangle $MHN$. Prove that $F J = F A$.
62 replies
Achillys
Jun 24, 2018
Rayvhs
an hour ago
Unsymmetric FE
Lahmacuncu   1
N an hour ago by ja.
Source: Own
Find all functions $f:\mathbb{R} \rightarrow \mathbb{R}$ that satisfies $f(x^2+xy+y)+f(x^2y)+f(xy^2)=2f(xy)+f(x)+f(y)$ for all real $(x,y)$
1 reply
Lahmacuncu
2 hours ago
ja.
an hour ago
find angle
TBazar   3
N an hour ago by TBazar
Given $ABC$ triangle with $AC>BC$. We take $M$, $N$ point on AC, AB respectively such that $AM=BC$, $CM=BN$. $BM$, $AN$ lines intersect at point $K$. If $2\angle AKM=\angle ACB$, find $\angle ACB$
3 replies
TBazar
6 hours ago
TBazar
an hour ago
Polynomial and Irrational Coefficients
Iamsohappy   1
N an hour ago by tom-nowy
Find all natural numbers $n$ such that there exists a monic polynomial of degree $n$ and with irrational coefficients ( excepts its leading term ) and such that it has $n$ distinct irrational roots.
1 reply
Iamsohappy
Jul 5, 2019
tom-nowy
an hour ago
Divisibility on 101 integers
BR1F1SZ   5
N 2 hours ago by Grasshopper-
Source: Argentina Cono Sur TST 2024 P2
There are $101$ positive integers $a_1, a_2, \ldots, a_{101}$ such that for every index $i$, with $1 \leqslant i \leqslant 101$, $a_i+1$ is a multiple of $a_{i+1}$. Determine the greatest possible value of the largest of the $101$ numbers.
5 replies
BR1F1SZ
Aug 9, 2024
Grasshopper-
2 hours ago
Nice number theory problem
ItsBesi   9
N 2 hours ago by Jupiterballs
Source:  Kosovo Math Olympiad 2025, Grade 8, Problem 3
Let $m$ and $n$ be natural numbers such that $m^3-n^3$ is a prime number. What is the remainder of the number $m^3-n^3$ when divided by $6$?
9 replies
ItsBesi
Nov 17, 2024
Jupiterballs
2 hours ago
My Unsolved Problem
ZeltaQN2008   1
N 3 hours ago by wh0nix
Let \(f:[0,+\infty)\to\mathbb{R}\) be a function which is differentiable on \([0,+\infty)\) and satisfies
\[
\lim_{x\to+\infty}\bigl(f'(x)/e^x\bigr)=0.
\]Prove that
\[
\lim_{x\to+\infty}\bigl(f(x)/e^x\bigr)0.
\]
1 reply
ZeltaQN2008
4 hours ago
wh0nix
3 hours ago
Find the value
sqing   8
N 3 hours ago by xytunghoanh
Source: Own
Let $a,b,c$ be distinct real numbers such that $ \frac{a^2}{(a-b)^2}+ \frac{b^2}{(b-c)^2}+ \frac{c^2}{(c-a)^2} =1. $ Find the value of $\frac{a}{a-b}+ \frac{b}{b-c}+ \frac{c}{c-a}.$
Let $a,b,c$ be distinct real numbers such that $\frac{a^2}{(b-c)^2}+ \frac{b^2}{(c-a)^2}+ \frac{c^2}{(a-b)^2}=2. $ Find the value of $\frac{a}{b-c}+ \frac{b}{c-a}+ \frac{c}{a-b}.$
Let $a,b,c$ be distinct real numbers such that $\frac{(a+b)^2}{(a-b)^2}+ \frac{(b+c)^2}{(b-c)^2}+ \frac{(c+a)^2}{(c-a)^2}=2. $ Find the value of $\frac{a+b}{a-b}+\frac{b+c}{b-c}+ \frac{c+a}{c-a}.$
8 replies
sqing
Mar 17, 2025
xytunghoanh
3 hours ago
Simple inequality
sqing   22
N 3 hours ago by ND_
Source: JBMO 2011 Shortlist A3
$\boxed{\text{A3}}$If $a,b$ be positive real numbers, show that:$$ \displaystyle{\sqrt{\dfrac{a^2+ab+b^2}{3}}+\sqrt{ab}\leq a+b}$$
22 replies
sqing
May 15, 2016
ND_
3 hours ago
greatest volume
hzbrl   0
3 hours ago
Source: purple comet
A large sphere with radius 7 contains three smaller balls each with radius 3 . The three balls are each externally tangent to the other two balls and internally tangent to the large sphere. There are four right circular cones that can be inscribed in the large sphere in such a way that the bases of the cones are tangent to all three balls. Of these four cones, the one with the greatest volume has volume $n \pi$. Find $n$.
0 replies
hzbrl
3 hours ago
0 replies
4 variables with quadrilateral sides 2
mihaig   6
N May 2, 2025 by mihaig
Source: Own
Let $a,b,c,d\geq0$ satisfying
$$\frac1{a+1}+\frac1{b+1}+\frac1{c+1}+\frac1{d+1}=2.$$Prove
$$\left(a+b+c+d-2\right)^2+8\geq3\left(abc+abd+acd+bcd\right).$$
6 replies
mihaig
Apr 29, 2025
mihaig
May 2, 2025
4 variables with quadrilateral sides 2
G H J
G H BBookmark kLocked kLocked NReply
Source: Own
The post below has been deleted. Click to close.
This post has been deleted. Click here to see post.
mihaig
7361 posts
#1 • 1 Y
Y by arqady
Let $a,b,c,d\geq0$ satisfying
$$\frac1{a+1}+\frac1{b+1}+\frac1{c+1}+\frac1{d+1}=2.$$Prove
$$\left(a+b+c+d-2\right)^2+8\geq3\left(abc+abd+acd+bcd\right).$$
Z K Y
The post below has been deleted. Click to close.
This post has been deleted. Click here to see post.
mihaig
7361 posts
#2
Y by
See, of course,
https://artofproblemsolving.com/community/c6t243f6h3555859_4_variables_with_quadrilateral_sides
Z K Y
The post below has been deleted. Click to close.
This post has been deleted. Click here to see post.
mihaig
7361 posts
#3
Y by
Let $a,b,c,d\geq0$ satisfying
$$\frac1{a+1}+\frac1{b+1}+\frac1{c+1}+\frac1{d+1}=2.$$Prove or disprove
$$\left(a+b+c+d-3\right)^2+7\geq2\left(abc+abd+acd+bcd\right).$$
Z K Y
The post below has been deleted. Click to close.
This post has been deleted. Click here to see post.
arqady
30240 posts
#4
Y by
mihaig wrote:
Let $a,b,c,d\geq0$ satisfying
$$\frac1{a+1}+\frac1{b+1}+\frac1{c+1}+\frac1{d+1}=2.$$Prove or disprove
$$\left(a+b+c+d-3\right)^2+7\geq2\left(abc+abd+acd+bcd\right).$$
It's wrong.
Try $(a,b,c,d)=\left(\frac{1}{2},\frac{5}{4},\frac{5}{4},\frac{5}{4}\right).$
Z K Y
The post below has been deleted. Click to close.
This post has been deleted. Click here to see post.
arqady
30240 posts
#5
Y by
mihaig wrote:
Let $a,b,c,d\geq0$ satisfying
$$\frac1{a+1}+\frac1{b+1}+\frac1{c+1}+\frac1{d+1}=2.$$Prove
$$\left(a+b+c+d-2\right)^2+8\geq3\left(abc+abd+acd+bcd\right).$$
Nice! There is a nice solution.
Z K Y
The post below has been deleted. Click to close.
This post has been deleted. Click here to see post.
mihaig
7361 posts
#6
Y by
arqady wrote:
mihaig wrote:
Let $a,b,c,d\geq0$ satisfying
$$\frac1{a+1}+\frac1{b+1}+\frac1{c+1}+\frac1{d+1}=2.$$Prove or disprove
$$\left(a+b+c+d-3\right)^2+7\geq2\left(abc+abd+acd+bcd\right).$$
It's wrong.
Try $(a,b,c,d)=\left(\frac{1}{2},\frac{5}{4},\frac{5}{4},\frac{5}{4}\right).$

Bravo!
Z K Y
The post below has been deleted. Click to close.
This post has been deleted. Click here to see post.
mihaig
7361 posts
#7
Y by
arqady wrote:
mihaig wrote:
Let $a,b,c,d\geq0$ satisfying
$$\frac1{a+1}+\frac1{b+1}+\frac1{c+1}+\frac1{d+1}=2.$$Prove
$$\left(a+b+c+d-2\right)^2+8\geq3\left(abc+abd+acd+bcd\right).$$
Nice! There is a nice solution.

Thank you!
Z K Y
N Quick Reply
G
H
=
a